Pages

Saturday, August 16, 2008

Pathology Mcqs

1.A patient complains of difficulty breathing through his nose and bony pain in his cheeks, near his nose. Physical examination and CT of the head reveal mass lesions involving the nose, pharynx, and sinuses. CT-guided biopsy demonstrates a non-keratinizing, squamous cell carcinoma. Which of the following disorders is associated with the same oncogenic virus that is the likely cause of this patient's cancer?

A. Adult T-cell leukemia

B. Burkitt's lymphoma

C. Cervical carcinoma

D. Hepatocellular carcinoma

E. Kaposi's sarcoma
Explanation:
The correct answer is B. The disease is nasopharyngeal carcinoma, which is associated with the Epstein-Barr Virus (EBV). This virus is also associated with the African form of Burkitt's lymphoma that characteristically involves the jaw.
HTLV-1, or human T-lymphocyte virus, is associated with adult T-cell leukemia (choice A).
HPV, or human papillomavirus, is associated with cervical carcinoma (choice C), penile carcinoma, and anal carcinoma.
Hepatitis B virus (HBV) is associated with hepatocellular carcinoma (choice D). HHV 8, a member of the herpes family, is associated with Kaposi's sarcoma (choice E).


Continue solving these wonderful set of Mcqs to deepen your Knowledge in Pathology


2.A 34-year-old man develops pulmonary hemorrhage and glomerulonephritis. Lung biopsy with immunofluorescence demonstrates IgG deposition along the basement membrane. These antibodies are most likely directed against which of the following types of collagen?

A. Type I

B. Type II

C. Type III

D. Type IV

E. Type X
Explanation:
The correct answer is D. The disease described is Goodpasture's syndrome, in which autoantibodies to basement membrane proteins cause damage to the lungs and kidneys. Pulmonary hemorrhage (especially in smokers) and rapidly progressive glomerulonephritis are common. The characteristic autoantibody present is directed against Type IV collagen, a component of the basement membrane.
Type I collagen (choice A) is found in bone, skin, tendon, dentin, fascia, and late wound repair.
Type II collagen (choice B) is found in cartilage (including hyaline cartilage), the vitreous body of the eye, and the nucleus pulposus of the intervertebral disk. Type III collagen (choice C) is found in skin, blood vessels, uterus, fetal tissue, and granulation tissue.
Type X collagen (choice E) is found in epiphyseal plates.

3.A patient has long-standing severe hemolytic anemia characterized by hypochromic cells. Electrophoresis studies demonstrate a near complete absence of beta chains. Several years later, the patient develops cardiac failure. Intracardiac deposition of which of the following would be most likely to contribute to the cardiac failure?

A. Calcium

B. Iron

C. Magnesium

D. Potassium

E. Sodium
Explanation:
The correct answer is B. The disease is beta thalassemia major, which is a severe hemolytic anemia characterized by a failure to produce the beta chains of hemoglobin (some HbF, the fetal form of hemoglobin, is produced). The excess alpha chains are insoluble, leading to intra- and extravascular hemolysis. These patients require large numbers of transfusions, and iron overload with resulting secondary hemochromatosis can contribute to eventual cardiac failure. The heart is also damaged by the chronic high output state needed to compensate for the anemia.
Calcium (choice A) deposition is seen in damaged tissues and states with high serum calcium, such as hyperparathyroidism.
Magnesium (choice C), potassium (choice D), and sodium (choice E) are highly soluble and do not usually precipitate in tissues.

4.A 30-year-old woman presents with complaints of weakness and headaches. Her friends say she has been irritable and depressed lately. On physical examination, the patient is jaundiced, and her liver is small and firm. Neurologic examination is remarkable for choreoathetotic movements and a fine tremor that, when her upper limbs are extended, resembles a bird flapping its wings. Which of the following tests would most likely lead to to correct diagnosis?

A. Nerve conduction studies

B. Prussian blue stain of liver biopsy specimen

C. Serum alkaline phosphatase

D. Serum transaminases

E. Slit-lamp examination of the eyes

Explanation:

The correct answer is E. This patient is exhibiting symptoms of Wilson's disease, which is due to inadequate copper excretion by the biliary system. Mutations in a copper-transporting ATPase, coded for by the ATP7B gene on chromosome 13, appear to underlie this autosomal recessive disorder. Accumulation of copper in the liver initially produces fatty change, followed by hepatocellular necrosis, inflammation, bile duct proliferation,
and cirrhosis. Eventually, the copper spills out of the liver to deposit in other tissues, notably the brain. The caudate nucleus and putamen are generally most affected; injury to these structures produces an extrapyramidal movement disorder that most commonly presents with choreoathetosis and tremor but which may produce cerebellar signs or parkinsonism. Various psychiatric symptoms can accompany neurological
involvement. Copper deposition in Descemet's membrane in the cornea produces the nearly pathognomonic Kayser-Fleischer ring, which can be seen with slit-lamp examination of the eyes. Failure to demonstrate a Kayser-Fleischer ring in a patient with hepatic disease and neurological impairment virtually excludes the diagnosis of Wilson's disease. Low serum ceruloplasmin and increased urinary copper, or increased copper
levels on liver biopsy, are diagnostic.

Nerve conduction studies (choice A) would be of little value in the diagnosis of Wilson's disease, although they are valuable in detecting dysfunction of peripheral nerves that result, for example, from demyelination, loss of nerve axons, failure of conduction, or neuromuscular junction failure.

A Prussian blue stain performed on a liver biopsy (choice B) would demonstrate increased iron stores in a patient's liver secondary to hemochromatosis.

Serum alkaline phosphatase (choice C) would be elevated in patients with biliary tract disease, although other types of liver disease, bone disease, or pregnancy can produce elevations as well.

Serum transaminases (choice D) would be increased in patients with a variety of diseases causing hepatocellular injury, including hepatitis and cirrhosis, but this finding is not particularly specific.

5.A physician in the emergency department is evaluating a patient with severe chronic obstructive pulmonary disease prior to oxygen supplementation. The physician decides to draw arterial blood for blood gas studies. His technique is faulty, however, and he introduces room air into the syringe while pulling on the plunger as he is drawing the
syringe out of the patient. Which of the following patterns of changes would be most likely to be produced by this exposure of arterial blood to room air?

PO2
PCO2
pH

A. Decreased
Decreased
Decreased

B. Decreased
Elevated
Decreased

C. Elevated
Decreased
Decreased

D. Elevated
Decreased
Elevated

E. Elevated
Elevated
Elevated

Explanation:

The correct answer is D. The technical part of the collection of arterial blood samples is difficult. Some hospitals allow only physicians to collect the samples, while other hospitals allow nurses or technicians with additional special training to collect the samples. No matter who performs the arterial draw, care must be taken to avoid
exposing the blood to room air, as such exposure tends to cause the blood to partially equilibrate with the room air.

Room air would have a higher PO2 and a lower PCO2 than this patient's blood, so the sample would have a
higher PO2 and a lower PCO2. In the atmosphere, PO2 = 150 mm Hg and PCO2 is near 0 mm Hg; in the arterial
blood of a healthy patient, PO2 = 100 mm Hg, PCO2 = 40 mm Hg (PO2 could be lower and PCO2 higher in a
diseased individual). Because CO2 is decreased, there will be less carbonic acid present in the blood, thus
raising the pH.

6.In which of the following sites do myxopapillary ependymomas most frequently occur?

A. Cerebellum

B. Conus medullaris

C. 4th ventricle

D. Lateral ventricles

E. Midbrain

Explanation:

The correct answer is B.

Myxopapillary ependymoma is a variant of ependymoma, a tumor arising from ependymal cells. Histologically, myxopapillary ependymoma contains a myxoid (mucus-rich) intercellular matrix, in which spindly neoplastic ependymal cells are arranged in a fascicular and papillary pattern (hence its designation). It is a benign tumor that almost always occurs in the distal segment of the spinal cord, ie, the
conus medullaris. Once excised, the patient is cured.

The cerebellum (choice A) is the favorite site for pilocytic astrocytomas, medulloblastomas, and hemangioblastomas, but not ependymomas.

In general, classic ependymomas occur in close proximity to the ventricular cavities, specifically, the 4th ventricle (choice C) in children and spinal cord in adults. The myxopapillary variant does not occur in either location.

A midbrain location (choice E) would be truly exceptional for any type of ependymoma.

7.A 54-year-old woman with chronic microcytic hypochromic anemia also has a sore, smooth, red tongue and a sense of dysphagia midway during swallowing. This patient is at increased risk for developing which of the following conditions?

A. Adenocarcinoma of the esophagus

B. Barrett's esophagus

C. Candida esophagitis

D. CMV esophagitis

E. Squamous cell carcinoma of esophagus

Explanation:

The correct answer is E.

The patient has Plummer-Vinson syndrome, characterized by atrophic glossitis,
esophageal webs, and iron-deficiency anemia. Patients with this syndrome are at increased risk of developing squamous cell carcinoma of the esophagus.

Barrett's esophagus (choice B) and adenocarcinoma of the esophagus (choice A) are associated with reflux
esophagitis.

Candida(choice C) and CMV (choice D) esophagitis can be seen in immunosuppressed patients, including AIDS
patients.

8.Which of the following locations is most likely for the development of carcinoma in a 32-year-old baseball player who has chewed tobacco for 15 years?

A. Floor of the mouth

B. Lower lip

C. Tongue

D. Tonsils

E. Upper lip

Explanation:

The correct answer is B.

Oral cancer is most strongly related to tobacco chewing, with weaker associations with
cigarette smoking, pipe smoking, and alcohol use. Oral cancers tend to occur on the

lower lip (40%; choice B), tongue (20%; choice C), floor of the mouth (15%; choice A), with other oral sites (choices D and E) being less common. They are usually
squamous cell carcinomas and unlike their skin counterparts, frequently cause both extensive morbidity and mortality.

9.A 67-year-old male smoker presents to his physician for a routine physical examination. Chest x-ray demonstrates a 2-cm density on the left side. Laboratory studies are remarkable for a serum sodium of 134 mEq/L. The findings may be attributable to tumor cell secretion of

A. adrenocorticotrophic hormone (ACTH)

B. antidiuretic hormone (ADH)

C. melanocyte-stimulating hormone (MSH)

D. parathyroid hormone (PTH)

E. vasoactive intestinal polypeptide (VIP)

Explanation:

The correct answer is B. All of the hormones listed can be secreted by bronchogenic carcinoma, and may cause
a paraneoplastic syndrome. Of the answer choices provided, only ADH (antidiuretic hormone) causes
hyponatremia.

ACTH (choice A) causes Cushing's syndrome.

MSH (choice C) causes increased skin pigmentation.

PTH (choice D) causes hypercalcemia.

VIP (choice E) causes diarrhea and hypokalemia.

Other hormones that can be produced include human chorionic gonadotropin (hCG; gynecomastia), prolactin
(lactation), and calcitonin (hypocalcemia).

10.A 48-year-old female presents to the doctor with lower back pain. She states that she has had the pain for about two weeks and that it has become steadily more severe. An x-ray shows a lytic bone lesion in her lumbar spine.Review of systems reveals the recent onset of mild headaches, nausea, and weakness. Her CBC shows a normocytic anemia, and her erythrocyte sedimentation rate is elevated. Urinalysis shows heavy proteinuria, and a serum protein electrophoresis shows a monoclonal peak of IgG. Which of the following is responsible for this patient's spinal lesion?

A. Bence-Jones proteins

B. Lymphoplasmacytoid proliferation

C. Osteoblast activating factor

D. Osteoclast activating factor

E. Primary amyloidosis (AL)

Explanation:

The correct answer is D. First of all, the disease described above is multiple myeloma.

Multiple myeloma is a plasma cell neoplasm in which the plasma cells proliferate a single, or monoclonal, type of immunoglobulin. In this case, and most commonly, IgG is produced. Patients with this disease are usually over 40 and may have normocytic anemias. They often complain of skeletal pain from lytic bone lesions and may report headaches
and nausea caused by hyperviscosity of the blood due to the excessive amounts of immunoglobulins. The lytic bone lesions are caused by the production of osteoclast activating factor by the neoplastic plasma cells. This can also lead to hypercalcemia.

Bence-Jones proteins (choice A), are immunoglobulin light chains. They are often overproduced in multiple myeloma and are filtered in the urine. They are not usually detected in serum unless there is renal impairment, but they can be detected in the urine by electrophoresis and immunofixation. They do not cause bony lytic lesions.

Lymphoplasmacytoid proliferation (choice B), describes a normal type of B lymphocyte which is morphologically between a lymphocyte and a plasma cell. Lymphoplasmacytoid lymphocytes produce IgM, and in Waldenstrom's macroglobulinemia, they undergo neoplastic proliferation and produce IgM peaks. Bone lesions are not seen in
this disease.

Osteoblast activating factor (choice C), would not produce osteolytic lesions and is not seen in multiple myeloma. There is a rare osteoblastic variant of multiple myeloma with dense bony osteosclerosis rather than lytic lesions, but osteoblast activating factor has not been shown to be involved.

Primary amyloidosis (AL) (choice E), is a primary light-chain type of amyloidosis associated with multiple myeloma. The insoluble proteinaceous deposits occur in the tongue, heart, kidney, and skin. This does not cause bony lytic lesions.

11.A 35-year-old man who recently traveled to a third world country develops chronic, severe dysentery.Colonoscopy demonstrates ulceration of the cecum, and a cecal biopsy reveals 15-to-40 micron amoebae with ingested erythrocytes and small nuclei with distinctive tiny central karyosomes. Which of the following organisms is the most likely culprit?

A. Acanthamoeba sp.

B. Balantidium coli

C. Entamoeba histolytica

D. Giardia lamblia

E. Naegleria fowleri

Explanation:

The correct answer is C.

Entamoeba histolytica is the usual cause of intestinal amebiasis, and has the
microscopic features described in the question stem. A particularly helpful (but not always present) feature of this organism is the presence of ingested red blood cells within the amoebae. These amoebae cause flask-shaped ulceration of the intestinal mucosa and submucosa, with a particular propensity for involving the cecum and ascending colon. The disease manifestations range from none (asymptomatic carriers) to mild chronic diarrhea, to severe, purging dysentery. In symptomatic cases, the liver may develop destructive
amoebic liver abscesses that tend to become secondarily (and potentially life-threateningly) infected by
bacteria.

Acanthamoeba(choice A) is a free-living amoebae that can cause amoebic meningocephalitis.

Balantidium coli(choice B) is a large ciliated intestinal parasite that can occasionally cause colonic disease resembling that caused by Entamoeba histolytica.

Giardia lamblia(choice D) is a small intestinal protozoa with a distinctive pear-shaped morphology that appears to have a "face."

Naegleria fowleri(choice E) is a free-living amoebae that can cause amoebic meningoencephalitis.

12.A 27-year-old man develops bilateral parotid gland swelling and orchitis, and is generally ill with fever of 102° F.Which of the following substances is most likely to be significantly elevated in the patient's serum?

A. Alanine aminotransferase (ALT)

B. Amylase

C. Aspartate aminotransferase (AST)

D. Ceruloplasmin

E. Creatine phosphokinase, MB isoenzyme (CPK-MB)

Explanation:

The correct answer is B.

The disease is mumps, caused by a paramyxovirus. In children, mumps causes a
transient inflammation of the parotid glands, and less commonly, the testes, pancreas, or central nervous system. Mumps tends to be a more severe disease in adults than in children. Mumps in adults involves the testes (causing orchitis) and pancreas with some frequency. Pancreatic involvement can cause elevation of serum amylase.

ALT (choice A) and AST (choice C) are markers for hepatocellular damage.

Ceruloplasmin (choice D) is a copper-carrying protein that is decreased in Wilson's disease.

CPK-MB (choice E) is the isoenzyme of CPK that is relatively specific for the myocardium. This enzyme is increased in the early stages of a myocardial infarction.

13.A 50-year-old hypertensive man develops very severe, "tearing" chest pain, which migrates from his upper back to mid-back over the period of an hour. Pathologic examination of a specimen removed from the patient during emergency surgery would most likely demonstrate which of the following?

A. Cystic medial degeneration

B. Infarction

C. Plasma cells around the vasa vasorum

D. Severe atherosclerosis

E. Tree-barking

Explanation:

The correct answer is A.

This is a classic description of a dissecting aortic aneurysm, a very important condition
that may cause death if missed or misdiagnosed. Dissecting aneurysms are actually dissecting hematomas, with the blood located between the middle and outer thirds of the media of the aorta. Dissecting aneurysms are associated with hypertension in many cases; they are also associated with cystic medial degeneration of the wall of the aorta (seen in Marfan's syndrome). Often, an intimal tear is present; these are thought to represent the starting point for the dissection. Unlike abdominal aortic aneurysm and syphilitic aneurysm, aortic dissection is not usually associated with aortic dilatation.

The pain of myocardial infarction (choice B) does not usually move.

Plasma cells around the vasa vasorum (choice C) and "tree-barking" (wrinkling of the aortic intima; choice E ) are features of syphilitic aneurysms.

Atherosclerotic (choice D) aneurysms typically affect the abdominal aorta.

14.The presence of the Philadelphia chromosome is associated with a more favorable prognosis in patients with
which of the following diseases?

A. Acute lymphoblastic leukemia

B. Acute myelogenous leukemia

C. Chronic lymphocytic leukemia

D. Chronic myelogenous leukemia

E. Hairy cell leukemia

Explanation:

The correct answer is D.

The presence of the Philadelphia chromosome, a translocation from the long arm of
chromosome 22 to chromosome 9 [t(9;22)], is associated with a more favorable prognosis in patients with chronic myelogenous leukemia.

Acute lymphoblastic leukemia (ALL; choice A) is the most common cause of leukemia in children. The presence of the Philadelphia chromosome is associated with a worse prognosis for the patient.

Acute myelogenous leukemia (AML; choice B) is the most common acute leukemia in adults. The M2 subtype is associated with the t(8;21) translocation and the M3 subtype is associated with the t(15;17) translocation.

Over half of patients with chronic lymphocytic leukemia (choice C) display one of several chromosomal abnormalities. This includes trisomy 12 (involves the h-ras proto-oncogene), translocation t(11;14) (involves
k-ras and bcl-1 proto-oncogenes), and deletion (14q-) or inversion (14q) (involves immunoglobulin heavy chain
gene).

Hairy cell leukemia (choice E) is associated with the expression of tartrate-resistant acid phosphatase (TRAP) on the surface of B cells.

15.A 32-year-old woman visits her gynecologist for a Pap smear. On physical examination, her gynecologist palpates a large adnexal mass on the right. After ultrasound confirmation of a large ovarian mass, a laparotomy is scheduled, and the mass is removed. Pathologic examination of the mass demonstrates a cystic cavity filled with hair and keratin debris, and the wall contains skin, adnexal tissue, thyroid tissue, and neural tissue. All of the
tissues are similar to those normally found, and no malignant changes are seen. Which of the following is the most likely diagnosis?

A. Immature teratoma

B. Leiomyoma

C. Leiomyosarcoma

D. Mature teratoma

E. Rhabdomyosarcoma

Explanation:

The correct answer is D.

The lesion is a mature teratoma. Teratomas located in the ovary and containing a hair
and keratin filled cyst are sometimes called dermoid cysts. Teratomas contain cells of a variety of types, often including skin, skin adnexal structures (hair follicles, sweat glands, sebaceous glands), connective tissues,neural tissue, muscle, and thyroid tissue.

If immature tissues such as primitive neuroepithelial cells or developing skeletal muscle cells are seen, the lesion is considered potentially malignant and classified as an
immature teratoma (choice A).

Leiomyomas (choice B) are benign tumors of smooth muscle (e.g., uterine "fibroids"), usually in the female genital tract.

Leiomyosarcomas (choice C) are rare malignant tumors of smooth muscle, usually in the female genital tract.

Rhabdomyosarcomas (choice E) are malignant skeletal muscle tumors with a predilection for the head and neck
and urogenital regions in children.

16.An endocrinologist examines a patient suspected of having Riedel thyroiditis. Which of the following findings on physical examination would best help confirm the diagnosis?

A. Eyeball protrusion

B. Massive soft thyroid gland

C. Single large thyroid nodule

D. Very tender and painful thyroid

E. "Woody" thyroid gland

Explanation:

The correct answer is E.

Riedel thyroiditis, also called ligneous (rocklike) stroma, is a rare form of chronic
thyroiditis characterized microscopically by a marked fibrous reaction that destroys most or all of the thyroid gland and may involve adjacent structures. The etiology is unknown. Clinically, this disease tends to affect
middle-aged and older, mostly female patients and causes the thyroid to have a firm "woody" texture. It may be
clinically mistaken for a neck malignancy and can cause symptoms of stridor, dyspnea, dysphasia, laryngeal
nerve paralysis, or hypothyroidism.

Eyeball protrusion (choice A) suggests the hyperthyroidism of Graves disease.

A massive, soft thyroid gland (choice B) suggests multinodular goiter.

A single large thyroid nodule (choice C) could be due to either a thyroid adenoma or thyroid cancer.

A very tender and painful thyroid (choice D) suggests subacute granulomatous (de Quervain) thyroiditis.

16.A 40-year-old woman with polycythemia vera develops progressive severe ascites and tender hepatomegaly over a period of several months. Liver function tests are near normal. Which of the following tests would be most likely to establish the diagnosis?

A. Endoscopic retrograde cholangiopancreatography (ERCP)

B. Hepatic venography

C. Serum alpha-fetoprotein

D. Serum ceruloplasmin

E. Serum iron studies

Explanation:

The correct answer is B.

The clinical presentation is most consistent with Budd-Chiari syndrome (hepatic vein
obstruction), which may occur as a complication of thrombogenic and myeloproliferative disorders, including polycythemia vera. The presentation illustrated is the most common; alternative presentations include fulminant
liver failure and cases in which intractable abdominal pain is the most prominent initial finding. The best method listed to establish the diagnosis of Budd-Chiari syndrome is hepatic venography to demonstrate the occlusion of
the hepatic venous system. Liver biopsy to provide evidence of centrilobular congestion and sinusoidal dilatation (in the absence of right-sided heart failure) is definitive, but more invasive.

Endoscopic retrograde cholangiopancreatography (choice A) is most useful in demonstrating lesions of the biliary tree.

Serum alpha-fetoprotein (choice C) is a marker for hepatocellular carcinoma.

Ceruloplasmin levels (choice D) are altered in Wilson's disease, in which cirrhosis and brain damage occur secondary to abnormalities in the metabolism of copper.

Serum iron studies (choice E) are useful when considering hemochromatosis as a cause of cirrhosis.

17.A 49-year-old homemaker presents to her physician because she noticed a lump in her breast during self-examination. Biopsy of the lump demonstrates invasive ductal carcinoma. The connective tissue adjacent to the nests of tumor is very densely collagenous. This is an example of which of the following processes?

A. Anaplasia

B. Carcinoma in situ

C. Desmoplasia

D. Dysplasia

E. Metaplasia

Explanation:

The correct answer is C.

This is an example of desmoplasia, which is excessive fibrous tissue formation in the
stroma of a tumor. The abundant fibrous tissue growth is, itself, benign.

Anaplasia (choice A) is a term used for tumors that show severe loss of cell differentiation and tissue organization; anaplastic tumors typically are much more clinically aggressive than their well-differentiated counterparts.

Dysplasia (choice D) is atypical cellular proliferation (without being so severe as to qualify for the diagnosis of cancer); an example is the epithelium seen in tubular adenomas of the colon.

In contrast, carcinoma in situ (choice B) is a similar change that is severe enough to be classified as cancer, but is confined to the epithelium with no invasion of underlying tissue. Carcinoma in situ can be found throughout the epithelial surfaces of the body.

Metaplasia (choice E) is the replacement of one type of differentiated cell or tissue by another not normally present at that site; an example is the replacement, in smokers, of the normal, ciliated, columnar epithelium of the respiratory tract with squamous epithelium.

18.Autopsy of an elderly individual who died in a nursing home with no known genetic diseases reveals small amounts of amyloid deposition in the heart. Amyloid deposition is not seen in other organs. There is no history of long-standing inflammatory disease. This type of amyloid would be most likely to be composed of which of the following proteins?

A. Amyloid-associated protein

B. Amyloid light chain protein

C. Beta-2-amyloid protein

D. Beta-2-microglobulin

E. Transthyretin

Explanation:

The correct answer is E.

This patient has senile cardiac amyloidosis, which is usually a clinically insignificant
condition due to deposition of structurally normal transthyretin (formerly called prealbumin). The transthyretin is a normal serum protein used to transport thyroxin and retinal. In addition to causing senile cardiac amyloidosis,
transthyretin, in a mutant rather than normal form, is deposited as amyloid in the familial amyloid polyneuropathies.

Amyloid-associated protein (choice A) is a more common protein deposited as amyloid, and precipitates in secondary amyloidosis associated with underlying chronic inflammatory conditions.

Amyloid light chain protein (choice B) is a common protein deposited as amyloid, and precipitates in amyloidosis related to multiple myeloma and other monoclonal B cell proliferations.

Beta-2-amyloid protein (choice C) is deposited as amyloid in the brain (notably in blood vessels and cerebral plaques) of patients with Alzheimer's disease.

Beta-2-microglobulin (choice D) is a normal serum protein that is deposited in amyloidosis complicating long-term hemodialysis.

19.Routine physical examination of a patient demonstrates proteinuria by dipstick method. No glucose is detected.Urine protein electrophoresis demonstrates a monoclonal spike. A tumor of which cell line would most likely
produce these findings?

A. Plasma cells

B. Renal tubular cells

C. Smooth muscle

D. T-lymphocytes

E. Transitional epithelium

Explanation:

The correct answer is A.

The tumor is multiple myeloma, a neoplasm of plasma cells. The monoclonal spike on
urine protein electrophoresis is typically due to excess light chains (associated with urinary Bence-Jones protein).

Monoclonal proteinuria would not be produced by tumors of the the other cell lines listed.

Renal cell carcinoma is a tumor of renal tubular cells (choice B).

Leiomyoma and leiomyosarcoma are tumors of smooth muscle (choice C).

Cutaneous T-cell lymphomas and T-cell leukemia are tumors of T-lymphocytes (choice D).

Transitional cell carcinoma is a tumor of transitional epithelium (choice E).

20.An 83-year-old female has a biopsy of an ulcerated nipple lesion that is interpreted as Paget's disease. A biopsy of the underlying breast tissue will most likely show which of the following?

A. Acute mastitis

B. Ductal carcinoma in situ

C. Intraductal papilloma

D. Invasive lobular carcinoma

E. Normal breast tissue

Explanation:

The correct answer is B.

Paget's disease of the breast is a form of ductal carcinoma in which neoplastic cells
involve the squamous epithelium of the skin by direct extension through the lactiferous ducts. Underlying breast tissue shows the origin of the ductal carcinoma-usually ductal carcinoma in situ and less frequently invasive ductal carcinoma.

Acute mastitis (choice A) is a disease of nursing women in which bacteria gain entry to the breast tissue via cracks in the traumatized nipple. It is characterized by acute inflammation and tissue necrosis.

Intraductal papilloma (choice C), a papillary mass arising within the ducts, usually presents as a single subareolar tumor that may produce a bloody or serous nipple discharge. Most intraductal papillomas are benign and are cured with complete excision.

Invasive lobular carcinoma (choice D) is a tumor of the terminal ductules of the breast. It presents as a poorly circumscribed, rubbery breast mass, unlike invasive ductal carcinoma, which tends to appear as a hard, stellate, and fibrous tumor. Lobular carcinoma does not produce Paget's disease.

Paget's disease of the breast always reflects underlying duct cancer. This is in marked distinction from extramammary Paget's disease, which may arise without an identifiable underlying malignancy (choice E).

21.A 52-year-old male presents with epigastric pain that improves with meals. Endoscopy demonstrates a 2 cm ulcerated area located 3 cm distal to the pyloric junction. Basal acid output is within normal limits. Which of the following is most likely to have made the strongest contribution to the development of this disease?

A. Aspirin use

B. Chronic antacid use

C. Drinking alcohol

D. Helicobacter pylori infection

E. Smoking

Explanation:

The correct answer is D.

The patient has a duodenal peptic ulcer. The strongest risk factor for duodenal peptic
ulcer is Helicobacter pylori infection, which is found in almost 100% of these cases (contrast to 70% infection rate in gastric peptic ulcer). The basal acid output is normal in many patients with duodenal ulcer.

Aspirin use (choice A) and ethanol use (choice C) are more strongly implicated in gastric ulcer disease than duodenal ulcer disease.

Chronic antacid use (choice B) is seen as a result of peptic ulcer disease, not as a cause of it.

Smoking (choice E) may also be a lesser contributing factor to the development of peptic ulcer.

22.A 42-year-old woman is noted to have mildly elevated creatinine and blood urea nitrogen on routine physical exam. She recalls that her father also had kidney trouble and died in kidney failure. Workup reveals persistent azotemia and microscopic hematuria without evidence of urinary tract infection. An ultrasound of the kidneys identifies bilaterally enlarged and multicystic kidneys. In addition to chronic renal failure, the clinician should also be concerned about her risk of

A. liver failure

B. pancreatic insufficiency

C. portal hypertension

D. renal cell carcinoma

E. subarachnoid hemorrhage

Explanation:

The correct answer is E.

Multicystic kidneys, slowly progressive renal failure, and a positive family history are
characteristics of autosomal dominant (adult) polycystic kidney disease (APKD). This disease typically presents in the 40s to 60s and is characterized by marked renal enlargement due to numerous fluid-filled cysts, which develop between the normally functioning nephrons.

APKD is highly associated with hepatic cysts, and berry
aneurysms in the circle of Willis that may rupture, producing spontaneous subarachnoid hemorrhage. Infrequently, APKD also produces cysts in the pancreas, spleen, or lungs, but these are not clinically relevant.

Hepatic cysts in adult polycystic disease do not ordinarily produce symptoms of hepatic failure (choice A).

Pancreatic cyst formation in APKD is not generally associated with pancreatic insufficiency (choice B).

Children with autosomal recessive polycystic kidney disease may develop congenital hepatic fibrosis with hypertension and splenomegaly, but this is not part of APKD (choice C).

APKD is not considered a risk factor for renal cell carcinoma or any other type of cancer (choice D).

23.A 32-year-old African American female presents with pelvic pain, low back pain, and a sensation of "pulling" or "stretching" in her groin. Bimanual examination reveals a firm mass in the right adnexa. An ultrasound examination reveals the presence of fluid in the abdominal cavity and the right thoracic cavity. Which of the following conditions is most strongly suggested by this patient's presentation?

A. Ectopic pregnancy

B. Endometrial implant

C. Ovarian fibroma

D. Pelvic inflammatory disease

E. Uterine leiomyoma

Explanation:

The correct answer is C.

Meigs syndrome is the unusual combination of hydrothorax (often right sided), ascites,
and an ovarian tumor (often a benign fibroma). Low back pain and a stretching or pulling sensation are characteristic of ascites in some patients (ascites can also be asymptomatic). The etiology of the fluid accumulation in Meigs syndrome remains a mystery.

Ectopic pregnancy (choice A) would not be associated with hydrothorax. If an ectopic pregnancy ruptured, blood could accumulate in the abdominal cavity, but the patient would likely be hypotensive or dead.

An endometrial implant (choice B) could cause pain, or impair fertility, but would not be expected to produce the combination of ascites and isolated right-sided hydrothorax.

Pelvic inflammatory disease (choice D) would be unlikely to produce the combination of ascites and isolated right-sided hydrothorax, although it might cause low back pain or pelvic pain.

A uterine leiomyoma (choice E) would not be expected to produce an adnexal mass, nor would it be likely to cause ascites and hydrothorax.

24.A 3-year-old child develops severe generalized edema following a viral infection. On the basis of clinical chemistry tests, a renal biopsy is performed, with normal light microscopic findings. Which of the following abnormal laboratory values might be expected in this individual?

A. Decreased alpha globulin levels

B. Decreased fibrinogen

C. Increased serum calcium levels

D. Low serum albumin levels

E. Red blood cell casts in the urine

Explanation:

The correct answer is D.

This child has minimal change disease, which is the major cause (over 90% of cases)
of nephrotic syndrome in children aged 2 to 6 years. The most prominent clinical chemistry finding in these patients is massive proteinuria. The urinary protein in minimal change disease, in contrast to other causes of nephrotic syndrome, is often composed predominantly of albumin. Many other clinical chemistry changes may
also be seen, including decreased serum albumin levels, hyperlipidemia, increased serum levels of alpha2- and
beta-globulins, decreased IgG, and increased fibrinogen.

Minimal change disease characteristically shows normal or near normal appearance of the glomeruli by light microscopy and extensive fusion of foot processes of the glomerular podocytes by electron microscopy. A point not always recognized by beginners is that the
podocyte alterations may represent a reaction to, rather than a cause of, the proteinuria (e.g., an attempt to "seal the holes" in the glomerulus), since varying degrees of foot process fusion (together with more specific features) may sometimes be seen in other glomerular diseases associated with the nephrotic syndrome.

Alpha-globulin levels (choice A) would be increased, rather than decreased, in minimal change disease.

Fibrinogen levels are increased, rather than decreased (choice B).

Serum calcium levels (choice C) are typically decreased in the nephrotic syndrome, possibly due to renal loss of vitamin D binding protein.

Red blood cell casts in the urine (choice E) are indicative of glomerulonephritis, rather than the nephrotic syndrome.

25.A 42-year-old African-American man sustains severe injuries in an automobile accident and is admitted to the intensive care unit. Examination of a peripheral blood smear on the 3rd day of admission reveals helmet cells, schistocytes, and decreased platelets. Which of the following is most strongly suggested by these findings?

A. Autoimmune hemolysis

B. Disseminated intravascular coagulation (DIC)

C. Hereditary spherocytosis

D. Megaloblastic anemia

E. Sickle cell anemia

Explanation:

The correct answer is B.

The findings suggest disseminated intravascular coagulation (DIC), which is a feared
complication of many other disorders, such as obstetrical catastrophes, metastatic cancer, massive trauma, and bacterial sepsis.

The basic defect in DIC is a coagulopathy characterized by bleeding from mucosal surfaces, thrombocytopenia, prolonged PT and PTT, decreased fibrinogen level, and elevated fibrin split products.
Helmet cells and schistocytes (fragmented red blood cells) are seen on peripheral blood smear.

Autoimmune hemolysis (choice A) and hereditary spherocytosis (choice C) would be characterized by spherocytes in the peripheral smear.

Macro-ovalocytes and hypersegmented neutrophils can be seen in megaloblastic anemia (choice D).

Sickle cells are seen in sickle cell anemia (choice E).

26.A 52-year-old man is found dead in his home. Autopsy reveals hemopericardium secondary to ventricular wall rupture. Roughly how long before his death did the man probably have a myocardial infarction?

A. 2 days

B. 7 days

C. 12 days

D. 20 days

E. 60 days

Explanation:

The correct answer is B.

Unsuspected (or denied) myocardial infarction is not uncommon, and death may occur
because of untreated complications. A number of serious complications can occur between 5 and 10 days following infarction, due to marked weakening of the necrotic myocardium. These include rupture of the ventricular wall leading to hemopericardium and cardiac tamponade (as this patient had), rupture of the interventricular septum, and rupture of the papillary muscle.

Arrhythmias are the most common complication 2 days post-infarction (choice A).

Fibrinous pericarditis secondary to an autoimmune phenomenon (Dressler's syndrome) can be seen several weeks after infarctions (choices C and D).

By 60 days after infarction (choice E), the contracted scar is usually complete, and residual complications include left ventricular failure and arrhythmias.

27.A 25-year-old man presents to a rheumatologist with complaints of joint pain involving the large joints of the legs.On questioning, the patient indicates that exacerbations in the joint pain are frequently accompanied by diarrhea.Which of the following gastrointestinal diseases is most likely to be implicated as the cause of the patient's joint problems?

A. Amebic colitis

B. Chronic appendicitis

C. Diverticulosis

D. Pseudomembranous colitis

E. Ulcerative colitis

Explanation:

The correct answer is E.

Several gastrointestinal diseases are associated with rheumatologic complaints. The
most frequent of these are the chronic inflammatory bowel diseases, ulcerative colitis and Crohn's disease,which can be associated with sacroiliitis (related to HLA-B27) or lower limb arthritis.

Other GI diseases associated with arthropathy include bypass surgery, Whipple's disease, Behcet's syndrome, and celiac disease.

Amebic colitis (choice A) is caused by ingestion of infectious cysts (typically from Entamoeba histolytica).Symptoms include abdominal pain and diarrhea; malaise and weight loss may occur. Cecal amebiasis can resemble acute appendicitis.

Chronic appendicitis (choice B) may be asymptomatic or cause poorly defined abdominal pain.

Diverticulosis (choice C) is usually a disease of older adults. It is often asymptomatic unless inflammation supervenes.

Pseudomembranous colitis (choice D) is a severe form of diarrhea usually seen in the setting of prior antibiotic use. The causative organism is almost always Clostridium difficile.

28.A 45-year-old woman complains of difficulty speaking, chewing, and swallowing. She experiences generalized weakness that increases with effort and as the day goes on. Symptoms are significantly improved after taking neostigmine. Autoantibodies responsible for causing the patient's condition are directed against

A. acetylcholine receptors

B. double-stranded DNA

C. dystrophin

D. erythrocyte surface antigens

E. myelin

Explanation:

The correct answer is A.

The patient has myasthenia gravis (MG), which typically produces weakness
worsening over the course of the day. It often affects the eye muscles and can produce diplopia. Neostigmine, an acetylcholinesterase inhibitor, would temporarily improve the patient's condition, which is associated with antibodies against nicotinic acetylcholine receptors present on skeletal muscle.

Antibodies to double-stranded DNA (choice B) as well as anti-Smith antibodies are found specifically in systemic lupus erythematosus. Peripheral nuclear staining is observed on immunofluorescence.

Antibodies to dystrophin (choice C) are not a recognized pathology. Abnormal or absent dystrophin, resulting from mutations in the X chromosome, is associated with Becker's and Duchenne muscular dystrophy, respectively. Pelvic girdle weakness and ataxia are classic symptoms.

Antibodies to erythrocyte surface antigens (choice D) can be found in warm antibody autoimmune hemolytic anemia. Patients with this condition would have a positive direct Coomb's test.

Antibodies to myelin (choice E) may play a role in multiple sclerosis, which is presumed to be of autoimmune etiology. This demyelinating disease is characterized by the spontaneous appearance and remission of
symptoms such as hyperreflexia, weakness, spasticity, dysarthria, tremor, ataxia, and visual disturbances.
Neostigmine would not produce any improvement.

30.A 10-year-old boy develops an itchy, vesicular rash, which is maximal on his face and trunk. Physical examination demonstrates a mixture of lesions, with macules, papules, vesicles, and crusted lesions. The mother reports that the lesions seem to be occurring in crops. Which of the following is the most likely diagnosis?

A. Herpes simplex I

B. Herpes simplex II

C. Measles

D. Shingles

E. Varicella

Explanation:

The correct answer is E.

This is varicella (chicken pox), which is the primary form of infection by the herpes
zoster (varicella-zoster) virus. Recurrence due to virus harbored in neurons tends to be dermatomal in distribution and is called shingles. Fever, malaise, headache, and myalgia may also be present, particularly in the prodromal phase.

Tzanck smear of the base of a vesicle may demonstrate multinucleated giant cells.
Immunocompromised patients can be treated with acyclovir to prevent dissemination. Chicken pox may be complicated by secondary bacterial infection, pneumonia, systemic spread (immunosuppressed patients),
neurologic involvement (rare), Reye's syndrome (rare), and hemolytic anemia (rare).

Herpes simplex I (choice A) causes oral vesicles and ulcers.

Herpes simplex II (choice B) causes genital vesicles and ulcers.

Measles (choice C) causes a blotchy, nonvesicular rash.

Shingles (choice D) is the recurrent form of herpes zoster infection and usually is localized to a single dermatome.

31.Which of the following complications is currently the major limitation to the long-term success of cardiac
transplantation?

A. Allograft rejection

B. Graft arteriosclerosis

C. Graft atherosclerosis

D. Opportunistic infections

E. Lymphoma

Explanation:

The correct answer is B.

Currently, graft arteriosclerosis (AKA graft vascular disease) is the most important limit
to the long-term success of heart transplantation. For unknown reasons, the coronary arteries of transplanted hearts undergo intimal thickening associated with hyperplasia of myocytes and fibroblasts and deposition of matrix. This results in luminal stenosis and myocardial ischemia.

Patients may develop myocardial infarction,which is clinically silent because the heart is denervated. The overall survival after heart transplantation is 80% at 1 year and 60% at 5 years. Do not confuse graft arteriosclerosis with graft atherosclerosis (choice C).
Atherosclerosis is caused by accumulation of cholesterol esters and development of atheromas.Atherosclerosis may recur in the coronary arteries of transplanted hearts, but is not a limiting factor in long-term success of heart transplantation.

Allograft rejection (choice A) is certainly a major postoperative problem. However, thanks to early diagnosis based on periodic endomyocardial biopsy and the availability of immunosuppressant therapy, this complication can be prevented or successfully treated.

Although opportunistic infections (choice D) and development of Epstein-Barr related lymphomas (choice E) are undesired effects of profound immunosuppression, these complications do not constitute a significant limitation to the overall outcome of cardiac transplantation.

32.A mother takes her 4-year-old to a pediatrician because the child is having chronic, severe headaches. Physical examination demonstrates poor visual tracking with one eye, which had not been present 1 year previously. The pediatrician orders a CT scan of the head, which demonstrates a cystic 4-cm mass above the pituitary gland.Resection of the tumor reveals a cystic lesion filled with dark, oily fluid containing granular debris. Histologic
examination of the tumor would most likely demonstrate a tumor with areas resembling which of the following?

A. Autonomic ganglion

B. Brain

C. Skin

D. Thyroid

E. Tooth enamel organ

Explanation:

The correct answer is E.

The tumor is a craniopharyngioma, which is also called an ameloblastoma because of
its histologic resemblance to tooth enamel organ, which contains ameloblasts. The resemblance is not merely coincidental, because the embryologic development of the pituitary involves both downward growth from the brain, forming the posterior lobe of the pituitary, and upward growth from the mouth (from remnants of Rathke's pouch), forming the anterior lobe of the pituitary. Craniopharyngiomas may occupy the sella turcica or may be found in a suprasellar location, often in the hypothalamus. The tumor may present with mass effects (as in this child) or pituitary insufficiency.

Ganglioneuromas contain tissue resembling autonomic ganglia (choice A).

Dermoid cysts (mature teratomas) of the ovary can contain tissue resembling brain (choice B), skin (choice C),or thyroid (choice D).

32.A 67-year-old male develops severe chest pain. He is admitted to the hospital, and diagnosed with a myocardial infarction based on his electrocardiogram and serial CK-MB levels. One week later, he again complains of precordial pain and develops a fever of 102°F (38.9°C). Physical examination is remarkable for a loud friction rub. Which of the following is the most likely diagnosis?

A. Caseous pericarditis

B. Fibrinous pericarditis

C. Hemorrhagic pericarditis

D. Purulent pericarditis

E. Serous pericarditis

Explanation:

The correct answer is B. Different types of pericarditis can be seen in different settings. Fibrinous and serofibrinous pericarditis may follow acute myocardial infarction (Dressler's syndrome) and can be seen in uremia, chest radiation, rheumatic fever, systemic lupus erythematosus, and following chest trauma (including chest surgery).

Caseous pericarditis (choice A) is generally due to tuberculosis.

Hemorrhagic pericarditis (choice C) can be seen with tuberculosis, malignant tumors, patients with bleeding diatheses, and following chest surgery.

Purulent pericarditis (choice D) is seen when pyogenic infections involve the pericardium, e.g., after cardiothoracic surgery.

Serous pericarditis (choice E) is seen in non-infectious inflammations (rheumatic fever, lupus, scleroderma,tumors, uremia).

33.A 25-year-old man presents with bilateral hearing loss. MRI reveals bilateral tumors within the cerebellopontine angles. Surgery is performed, and the tumors are removed. Both are found to be neurilemomas ("schwannoma").Which of the following is the most likely diagnosis?

A. Metastatic disease

B. Multiple sclerosis

C. Neurofibromatosis type 1

D. Neurofibromatosis type 2

E. Tuberous sclerosis

Explanation:

The correct answer is D.

Neurofibromatosis type 2 is an autosomal dominant condition caused by mutations of a
gene on chromosome 22 coding for a cytoskeleton-related protein called merlin. Much less common than neurofibromatosis type 1, it manifests with multiple CNS tumors,

the most frequent of which are schwannomas of the 8th cranial nerve and meningiomas. Bilateral schwannomas are virtually pathognomonic (ie, diagnostic) of neurofibromatosis type 2.

Metastases to the CNS (choice A) are often multiple and usually involve the gray-white matter junction. Besides the unusual location, the young age would make this diagnosis highly improbable.

Multiple sclerosis (choice B) is a chronic remitting/relapsing demyelinating disease. It manifests with focal neurologic deficits caused by well-circumscribed areas of myelin loss in the white matter of the brain (usually periventricular), brainstem, spinal cord, or optic nerves. It is not associated with an increased incidence of any
type of brain tumor.

Neurofibromatosis type 1 (choice C) is also autosomal dominant and is caused by mutations of a gene on chromosome 17 coding for neurofibromin, a protein involved in signal transduction.

The most characteristic clinical features include café-au-lait spots, neurofibromas (tumors of peripheral nerves different from schwannomas), Lisch nodules (pigmented nodules of the iris), and CNS tumors, but not schwannomas.

Tuberous sclerosis (choice E), like neurofibromatosis, is a "neurocutaneous syndrome," ie, a condition characterized by concomitant neurologic and skin lesions. Tuberous sclerosis is caused by mutations in two loci,either TS1 or TS2. Multiple hamartomas of the brain (cortical "tubers") and other organs, shagreen patches,ash-leaf patches, and other skin lesions constitute the clinical findings in this disorder.

34.A 35-year-old man is referred to a psychiatrist because of erratic behavior. The man had been adopted in infancy, so his family history is not known. Over the next year, he develops uncontrollable erratic movements,such that attempts to pick up a cup or use a pencil produce sudden uncontrolled lurches. When he tries to walk,he staggers, thrusts, and abruptly changes direction. Eventually, with disease progression, he develops increasing rigidity and is unable to move, and finally dies ten years after the onset of symptoms. Which of the following changes would most likely be seen on examination of his brain at autopsy?

A. Depigmentation of the substantia nigra and locus ceruleus

B. Diffuse cortical atrophy with relative sparing of primary motor and sensory areas

C. Selective frontal and temporal lobe atrophy

D. Striking degeneration of the caudate nucleus

E. Widespread neuronal loss and gliosis in subcortical sites

Explanation:

The correct answer is D.

The disease is autosomal dominant Huntington's chorea. The question stem describes
a typical clinical progression (the family history is usually strikingly positive).

Pathological findings include severe atrophy of the caudate nucleus (with loss of medium-sized spiny neurons), less severe involvement of the putamen and cerebral cortex, and dilation of the lateral ventricles apparent on CT and MRI studies. The disorder is known to be caused by expansion of a CAG trinucleotide repeat in a gene on the short arm of
chromosome 4 coding for a protein called huntingtin. There is no effective therapy.

Choice A is characteristic of Parkinson's disease, characterized by bradykinesia, pill-rolling tremor, and cogwheel rigidity.

Choice B is characteristic of Alzheimer's disease, a degenerative dementing disorder.

Choice C is characteristic of Pick's disease, a dementing disorder that may be confused with Alzheimer's disease. Microscopically, there is gliosis, neuronal loss, and swollen neurons, which may contain characteristic Pick bodies (silver-staining cytoplasmic inclusions).

Choice E is characteristic of progressive supranuclear palsy. Patients exhibit an extrapyramidal syndrome accompanied by dystonias of the neck and paralysis of downward gaze.

35.The presence of which of the following features in an atherosclerotic plaque indicates that it has become a complicated lesion?

A. Cholesterol crystals

B. Chronic inflammatory cells

C. Intimal smooth muscle

D. Lines of Zahn

E. Necrotic cell debris

Explanation:

The correct answer is D.

Complicated lesions indicate advanced atherosclerotic disease. They arise in
atherosclerotic plaques, and render them more susceptible to sudden occlusion and acute infarction of the supplied tissues. Commonly, the plaque ulcerates or ruptures, and the exposed surfaces, being highly thrombogenic, precipitate thrombus formation. T

hrombi are typified by the lines of Zahn, alternating layers of platelets and fibrin (the pale lines) and layers of blood (the dark lines). Beyond thrombus formation, other features of a complicated plaque include hemorrhage into the lesion itself, and microembolism by cholesterol crystals or calcified debris. Furthermore, the weakened media underlying the plaque may develop an aneurysmal dilatation. In general, the clinical significance of atherosclerosis is related to the consequences of complicated lesions.

The incorrect options all include features of atheromatous plaques, but do not indicate complicated lesions:

Beneath the endothelium of a plaque there is a fibrous cap composed of smooth muscle (choice C), chronic inflammatory cells (choice B) and lipid laden macrophages (foam cells), as well as extracellular material.

The core of the lesion, which lies between the intima and the media, is composed of necrotic cellular debris (choice E), with cholesterol crystals (choice A), calcium, and more foam cells.

36.A 72-year-old female patient with Alzheimer's disease, but no other medical problems, suddenly becomes comatose and dies due to an intracranial hemorrhage that caused severe damage to her entire left cerebral hemisphere. There was no evidence or history of trauma. What is the most likely cause of this hemorrhage?

A. Epidural hematoma

B. Subdural hematoma

C. Amyloid angiopathy

D. Rupture of berry aneurysm

E. Rupture of Charcot-Bouchard aneurysm

Explanation:

The correct answer is C.

Alzheimer's disease patients are prone to large "lobar" hemorrhages that are usually
centered in the parietal lobe (thus the name "lobar") and may spread to totally destroy an entire cerebral hemisphere, resulting in death. This is due to amyloid deposition into the walls of cerebral blood vessels (amyloid angiopathy), similar to the amyloid plaques seen in the parenchyma of the brain with this disease.
Amyloid makes these vessels weak and prone to rupture.

An epidural hematoma (choice A) is a collection of blood above the dura mater, usually due to a blow to the side of the head that fractures the temporal bone of the skull and shears the middle meningeal artery. Even though this fast-flowing arterial blood usually causes symptoms within the first 24 hours and can cause life-threatening mass effects, no traumatic event was involved in this case.

A subdural hematoma (choice B) forms when slow-flowing venous blood collects below the dura mater due to leakage from stretched cortical veins as they drain into the superior sagittal sinus. Even though it is true that Alzheimer's patients are more susceptible to these bleeds due to brain atrophy, causing cortical veins to be maximally stretched and prone to tearing, subdural hematomas are usually associated with some sort of trauma.
Furthermore, a subdural hematoma causes gradual symptoms over time, rather than presenting as a sudden devastating event as in this case.

Rupture of a berry aneurysm (choice D) usually causes a subarachnoid hemorrhage, in which blood leaks into the space between the arachnoid membrane and the brain. The blood may also gain access to the ventricles,
but does not generally damage the cerebral hemispheres. Subarachnoid hemorrhages can be sudden and deadly, but usually present with a severe headache ("the worst headache of my life"). These aneurysms are not especially associated with Alzheimer's disease.

Rupture of a Charcot-Bouchard aneurysm (choice E) might cause a sudden hemorrhage within the brain causing coma and death, but it is not especially associated with Alzheimer's disease. Charcot-Bouchard aneurysms are primarily seen in patients with severe hypertension.

37.A 37-year-old woman presents to her physician complaining of difficulty reading and fatigue. She reports having a "pins and needles" feeling in her left arm several months ago that resolved without treatment. On examination,visual field deficits and mild hyperreflexia are noted. MRI confirms the suspected diagnosis. Which of the following is the underlying mechanism of this patient's disease?

A. Antibodies to acetylcholine receptors

B. Axonal degeneration

C. Demyelination of the peripheral nerves

D. Loss of oligodendrocytes

E. Loss of Schwann cells

Explanation:

The correct answer is D.

This woman presents with the classic signs and symptoms of multiple sclerosis (MS). A
key to this disease is different neurological signs that are separated by space and time. (Another classic clue might have been oligoclonal bands on electrophoresis of the CSF.) MS is a demyelinating disease of the central nervous system, characterized by loss of oligodendroglial cells, which are the cells that are responsible for producing myelin in the central nervous system. Diagnosis can be confirmed by an MRI revealing sharply
delineated regions of demyelination (plaques) throughout the central nervous system white matter (especially in periventricular areas).

Antibodies to acetylcholine receptors (choice A) have been implicated in the etiology of myasthenia gravis, not multiple sclerosis.

MS is generally characterized by axonal preservation, rather than degeneration (choice B).

Demyelination of peripheral nerves (choice C) occurs in a number of diseases (e.g., Guillain-Barré), but not in MS. Guillain-Barré is characterized by ascending muscle weakness, areflexia, and paralysis.

Oligodendrocytes are responsible for producing myelin in the central nervous system; Schwann cells (choice E)are responsible for myelination in the peripheral nervous system, and are not affected in multiple sclerosis.

38.Which of the following endometrial lesions is associated with the highest risk of developing endometrial adenocarcinoma?

A. Chronic endometritis

B. Complex hyperplasia with atypia

C. Complex hyperplasia without atypia

D. Simple hyperplasia

E. Squamous metaplasia

Explanation:

The correct answer is B.

In general, any condition characterized by excessive estrogenic stimulation is
associated with some degree of endometrial hyperplasia and increased risk of endometrial cancer. Endometrial hyperplasia is a histologic precursor of endometrial adenocarcinoma. Hyperplasia may progress to invasive adenocarcinoma through progressive degrees of cellular and architectural atypia. The grade of hyperplasia,
therefore, is related to the severity of alterations of gland architecture, growth pattern, and cytologic features.
The most severe changes are present in complex hyperplasia with atypia. Disorganization and crowding of glands, high mitotic activity, and nuclear atypia characterize this change. Longitudinal studies show that 25% of women with this form of hyperplasia develop adenocarcinoma.

Complex hyperplasia without atypia (choice C) is characterized by similar crowding of glands and epithelial cells in the absence of cellular atypia: progression to cancer occurs in 5% of cases. In simple hyperplasia (choice D), the glands are dilated and irregular, but
cytologic atypia and mitoses are absent and transformation is rare.

Chronic endometritis (choice A) has no relationship with endometrial adenocarcinoma and refers to a condition of chronic inflammatory infiltration of the endometrium. A diagnosis of chronic endometritis is made only when plasma cells are found on biopsy. (Lymphocytes are normally present in the endometrial mucosa.) This condition is associated with pelvic inflammatory disease (PID), intrauterine contraceptive devices (IUD), and
retention of gestational tissue within the uterine cavity.

Squamous metaplasia (choice E) is occasionally found in endometrial biopsy and results from transformation of the normal columnar epithelium into squamous epithelium. It is not associated with endometrial adenocarcinoma.

39.A patient comes to medical attention after he sets off the metal detector at the airport, despite removing his watch, belt buckle, and every other obvious source of metal. Which of the following diseases might be responsible for this phenomenon?

A. Argyria

B. Gall stones

C. Hemochromatosis

D. Kidney stones

E. Wilson's disease

Explanation:

The correct answer is C.

Hemochromatosis is an iron storage disorder that can cause cirrhosis (with increased
risk of hepatocellular carcinoma), skin pigmentation, pancreatic damage leading to diabetes mellitus, and congestive heart failure. These complications are due to damage caused by deposition of iron in tissues; the total body iron in some of these individuals may reach 50 g, large enough to set off some airport metal detectors.

Argyria (choice A) is a blue-gray skin discoloration related to silver poisoning.

Neither gallstones (choice B) nor kidney stones (choice D) contain metal.

In Wilson's disease (choice E), copper is deposited in liver and brain, but this would not be detected by metal
detectors.

40.A Pap smear of a 23-year-old woman demonstrates squamous cells with enlarged, hyperchromatic nuclei and prominent perinuclear halos. The Pap smear is graded as cervical intraepithelial neoplasia, grade II (CIN II). Which of the following viruses is most likely to be etiologically related to this neoplastic growth?

A. Epstein-Barr virus (EBV)

B. Hepatitis B virus (HBV)

C. Human herpesvirus 8 (HHV8)

D. Human papillomavirus (HPV)

E. Human T-cell lymphotropic virus -1 (HTLV-1)

Explanation:

The correct answer is D.

CIN II corresponds to moderate dysplasia of the cervix. The CIN lesions of all grades
(including condyloma) and the cervical cancers that can arise from them appear to be associated with infection with certain subtypes of human papilloma virus (HPV). Koilocytotic atypia (enlarged, hyperchromatic nuclei and prominent perinuclear halos) is commonly observed with HPV infection. HPV is usually spread through sexual
contact and also causes penile and anal condyloma and carcinomas.

EBV (choice A) is associated with nasopharyngeal carcinoma and Burkitt's lymphoma.

HBV (choice B) is associated with hepatocellular carcinoma.

HHV8 (choice C) is associated with Kaposi's sarcoma.

HTLV-1 (choice E) is associated with adult T-cell leukemia.

41.A young man presents to his physician's office for a physical exam. He is concerned because his father died of a heart attack in his late 40's. The physician finds that he has elevated serum cholesterol and LDL levels, but his VLDL and triglycerides are normal. Further investigation reveals an LDL receptor deficiency. This patient has which of the following types of hyperlipidemia?

A. Type I

B. Type IIa

C. Type IIb

D. Type III

E. Type IV

F. Type V

Explanation:

The correct answer is B.

There are many clues in the question that should have guided you to this choice. The
laboratory findings are classic for Type IIa hyperlipidemia. These patients have LDL receptor deficiencies and are at a great risk of advanced coronary atherosclerosis. Since it is autosomal dominant, the patient's father could have been affected as well.

Type I hyperlipidemia (choice A), or familial hyperchylomicronemia, is caused by a lipoprotein lipase deficiency.These patients have high serum triglycerides and normal cholesterol. They do not have a substantially higher
risk of atherosclerosis.

Type IIb hyperlipidemia (choice C), or familial combined hyperlipidemia, presents as elevated serum LDL, VLDL,cholesterol, and triglycerides. These patients do have an increased incidence of atherosclerosis.

Type III hyperlipidemia (choice D), or familial dysbetalipoproteinemia, presents as increased serum cholesterol and triglycerides. The mode of inheritance is not understood, but apoprotein E is affected and the risk of atherosclerosis is great.

Type IV hyperlipidemia (choice E), or familial hypertriglyceridemia, presents as increased triglycerides with normal cholesterol and LDL. The disease may be sporadic and is possibly associated with an increased risk for atherosclerosis.

Type V hyperlipidemia (choice F), or mixed hypertriglyceridemia, is not common. Cholesterol is slightly increased and triglycerides are greatly increased. There is deficient apoprotein CII. The risk of atherosclerosis is not clear.

42.A pregnant woman develops deep, boring pain of her left thigh muscles associated with swelling and enhanced warmth of the same leg. The pain is worsened by extending the foot. The superficial veins of the leg are engorged. Her condition puts her at risk for which of the following?

A. Acute renal failure

B. Cerebral hemorrhage

C. Hepatic infarction

D. Myocardial infarction

E. Pulmonary embolus

Explanation:

The correct answer is E.

The patient has clinical findings strongly suggestive of deep venous thrombosis (DVT).
Pain that increases upon extension of the foot is referred to as Homans' sign. Pregnancy, particularly during the third trimester, induces a hypercoagulable blood state (possibly to limit the chance of fatal hemorrhage during delivery), which can manifest as DVT. The major complication of DVT is pulmonary embolism, which may be massive and can cause sudden death.

Renal failure (choice A) can be a serious problem in pregnancy, but is not related to deep vein thrombosis.

Cerebral hemorrhage (choice B) and hepatic infarction (choice C) are complications of preeclampsia.

The hypercoagulability that predisposes for deep vein thrombosis also predisposes for myocardial infarction (choice D), but MI is not considered a complication of DVT.

43.A 31-year-old African-American female dancer injures herself during a dance routine and is seen in the emergency room for tender ribs. A chest radiograph is performed which shows no rib fractures, but does reveal significant bilateral hilar adenopathy and middle and upper zone linear streaks and nodules. Upon further questioning she recalls an episode approximately one year ago, in which she experienced fever, ankle swelling,
and tender red bumps on her lower extremities. She currently has no medical complaints and is able to rehearse without any difficulty breathing. Which of the following is the most likely diagnosis?

A. No disease, the patient is healthy

B. Pneumocystis carinii pneumonia

C. Sarcoidosis

D. Systemic sclerosis

E. Wegener's granulomatosis

Explanation:

The correct answer is C.

This patient has sarcoidosis, a multi-system disease of unknown etiology characterized by noncaseating granulomas on histological examination of various organs. Sarcoidosis occurs mostly in blacks in the third to fifth decades. Approximately 90% of cases involve the lungs, with findings such as hilar adenopathy and pulmonary infiltration. Other findings include uveitis, erythema nodosum (the lesions on this patient's lower extremities), arthritis, central and peripheral neuropathies, cardiomyopathy, and hypercalcemia. Symptoms include fatigue, exertional dyspnea, and non-productive cough. Diagnosis is made after excluding other causes by examination of a tissue biopsy. Radiographic staging is performed by serial chest x-rays.

Pneumocystis carinii pneumonia (PCP) (choice B) is a disease almost exclusively of HIV-infected patients,usually with a characteristic chest radiograph appearance of bilateral fluffy infiltrates.

Systemic sclerosis (choice D), or scleroderma, is a systemic collagen-vascular disease that involves the skin nearly 100% of the time, and the esophagus in approximately 75% of patients.

Wegener's granulomatosis (choice E) is a disorder characterized by focal necrotizing vasculitis in the lung and upper airways associated with granuloma formation, and necrotizing glomerulitis.

44.Following a respiratory infection, a 20-year-old man goes to his physician for a follow-up visit. Physical examination is unremarkable, but dipstick analysis of his urine reveals marked proteinuria and microscopic hematuria. The young man is referred to a specialist, who performs a renal biopsy. Immunofluorescence microscopy of the biopsy tissue demonstrates IgA deposition in the glomerular mesangium. These results are
most consistent with which of the following disorders?

A. Berger's disease

B. Goodpasture's syndrome

C. Minimal change disease

D. Poststreptococcal glomerulonephritis

E. Systemic lupus erythematosus

Explanation:

The correct answer is A.

Berger's disease, or Ig A nephropathy, may develop after a respiratory infection. It is a
major cause of recurrent microscopic hematuria, and may progress to renal failure in a number of cases. (Note: don't confuse Berger's disease with Buerger's disease-a vasculitis that occurs in smokers.)

Questions about Goodpasture's syndrome (choice B) often contain a clue about linear deposition of IgG anti-basement membrane antibodies.

You should associate fusion of podocyte foot processes with minimal change disease (choice C).

Classic poststreptococcal glomerulonephritis (choice D) may follow pharyngitis caused by group A streptococcus. This type of glomerulonephritis is associated with granular immunofluorescence and subepithelial humps by electron microscopy.

Renal involvement in lupus (choice E) can have many manifestations, but there will usually be evidence of a systemic inflammatory disease.

45.A 63-year-old African-American male presents with vague complaints of abdominal and back pain, malaise,nausea, and weakness, which have been present for 3 or 4 months. Review of systems reveals a 15 pound weight loss, occasional vomiting, and several episodes of unilateral leg swelling, which have involved both legs at different times. These findings are most consistent with which of the following diagnoses?

A. Pancreatic cancer

B. Primary sclerosing cholangitis

C. Pyelonephritis

D. Reflux esophagitis

E. Splenic infarction

Explanation:

The correct answer is A.

Pancreatic carcinoma often presents with vague abdominal, back, and gastrointestinal
complaints; and physical examination is generally unrevealing. The tremendous weight loss is very suspicious for carcinoma, and the migrating thrombophlebitis (Trousseau's sign) is extremely helpful in making the diagnosis, which should be confirmed with ultrasonography or CT. Although any carcinoma can elicit migratory thrombophlebitis, it is mostly associated with tumors of the pancreas, lung, and colon.

Primary sclerosing cholangitis (choice B) is an inflammatory fibrosing disease of the biliary tree of unknown etiology, although highly associated with inflammatory bowel disease. It presents with symptoms of liver failure (jaundice, pruritus) and progresses to biliary cirrhosis.

Pyelonephritis (choice C) may present with back pain that usually localizes to the costovertebral angle, and is generally associated with fevers and dysuria.

Chronic pyelonephritis proceeds to hypertension and renal failure.

Reflux esophagitis (choice D) presents with heartburn, regurgitation, and dysphagia and may occasionally be confused with a heart attack by the patient. Long term consequences include bleeding, strictures, and Barrett's esophagus.

Splenic infarction (choice E), typically associated with arterial thromboembolic events, generally is an asymptomatic occurrence that does not produce clinical symptoms unless the entire spleen is lost.

46.A 37-year-old man presents to a physician because of a lesion on the shaft of his penis. On physical examination, a solitary, thickened, whitish plaque with a slightly ulcerated, crusted surface is observed. Biopsy reveals markedly dysplastic epithelial cells, many mitoses (some of which are abnormal), disordered epithelial maturation, and an intact basement membrane with no evidence of stromal invasion. Which of the following is the
most likely diagnosis?

A. Bowenoid papulosis

B. Bowen's disease

C. Condyloma acuminatum

D. Erythroplasia of Queyrat

E. Squamous cell carcinoma

Explanation:

The correct answer is B.

This is Bowen's disease, which is one clinical form of penile carcinoma in situ (the
other forms are Bowenoid papulosis and erythroplasia of Queyrat). Untreated Bowen's disease will, over a period of years, progress to frank invasive carcinoma in 10% to 20% of patients. Bowen's disease can affect the genital region of both men and women.

Bowenoid papulosis (choice A) is another form of penile carcinoma in situ, characterized clinically by multiple reddish-brown papular lesions.

Bowen disease Bowenoid papulosis
>35 years younger age of patients
solitary multiple lesions
thickened ,grey white, opaque plaque with shallow ulceration & crusting reddish brown papular lesions
10% progress into invasive carcinoma virtually never develops into invasive carcinoma

Condyloma acuminatum (choice C) usually produces a papillary lesion.

Erythroplasia of Queyrat (choice D) arises from the squamous epithelial cells of the glans penis or inner lining of the prepuce. It is seen almost exclusively in uncircumcised men and represents an in situ form of squamous cell carcinoma.

Squamous cell carcinoma (choice E) will be frankly invasive on biopsy.

47.A physician has been treating a 60-year-old patient with renal failure due to polycystic kidney disease. The physician should be specifically concerned about the possible coexistence of which of the following conditions?

A. Aneurysm of aortic root

B. Atherosclerotic aneurysm

C. Berry aneurysm

D. Cystic medial necrosis

E. Mycotic aneurysm

Explanation:

The correct answer is C. There is a specific association between the adult form of polycystic kidney disease and congenital berry aneurysms of the circle of Willis. These aneurysms can rupture, producing a subarachnoid hemorrhage, and possibly causing death.

Aneurysms of the aortic root (choice A) are associated with syphilis.

Atherosclerosis (choice B) causes abdominal aortic aneurysms, many of which are seen in diabetics.

Cystic medial necrosis (choice D) can cause dissecting aneurysms (e.g., in Marfan's syndrome).

"Mycotic" aneurysms (choice E) are due to bacterial, not fungal, infection.

48.A 50-year-old woman with a swan-neck deformity of the hands and enlarged knuckles develops large subcutaneous nodules near her elbows. If the nodules were biopsied, which of the following best describes their likely histological appearance?

A. Amorphous crystalline mass surrounded by macrophages

B. Cystic space caused by myxoid degeneration of connective tissue

C. Darkly pigmented synovium with an exuberant, villous growth

D. Fibrinoid necrosis surrounded by palisading epithelioid cells

E. Well-encapsulated nodule of polygonal cells within a tendon sheath

Explanation:

The correct answer is D.

Swan-neck deformity, enlarged knuckles, and subcutaneous nodules are classic clues
for rheumatoid arthritis. The subcutaneous rheumatoid nodules are composed histologically of areas of fibrinoid necrosis surrounded by palisading epithelioid cells.

Gout tophi are amorphous crystalline masses surrounded by macrophages (choice A).

Ganglion cysts are small cystic spaces caused by myxoid degeneration of connective tissue (choice B).

Pigmented villonodular synovitis causes darkly pigmented synovium with exuberant villous growth (choice C).

Nodular tenosynovitis causes a well-encapsulated nodule of polygonal cells within a tendon sheath (choice E).

49.A 21-year-old female with a several month history of easy bruising and increased menstrual flow is evaluated for a bleeding disorder. Her platelet count is 31,000/mm3. Subsequent investigations determine that she has idiopathic thrombocytopenic purpura (ITP). In this disorder, the low platelet count is due to which of the following?

A. Antiplatelet antibodies

B. Defective platelet aggregation

C. Hypersplenism

D. Ineffective megakaryopoiesis

E. Mechanical trauma

Explanation:

The correct answer is A.

ITP is a chronic autoimmune disorder in which antibodies against platelet glycoproteins
cause platelet destruction and removal by the reticuloendothelial system. Secondary thrombocytopenia can also be produced by lupus, viral infections, and drugs. Only when secondary thrombocytopenia has been ruled out, can the diagnosis of ITP be made.

Defective platelet aggregation (choice B) is responsible for thrombasthenia, an autosomal dominant disease that causes prolonged bleeding time but normal numbers of platelets.

Hypersplenism (choice C) causes thrombocytopenia when an enlarged spleen traps normal platelets in the absence of other specific platelet disorders. This type of thrombocytopenia can be cured with splenectomy. Although the thrombocytopenia in ITP often improves with splenectomy,

ITP does not cause splenomegaly.

Megakaryopoiesis (choice D) is disturbed in any disorder that causes bone marrow failure, including drug toxicity, leukemia, and infections. In this setting, thrombocytopenia is often part of a pancytopenia.

Mechanical trauma (choice E) causes thrombocytopenia in disseminated intravascular coagulation (DIC),thrombotic microangiopathies (thrombotic thrombocytopenic purpura and hemolytic-uremic syndrome), and with giant hemangiomas.

50.A patient with prolonged diarrhea undergoes esophagogastroduodenoscopy. Biopsy of the small intestine demonstrates numerous crescent-shaped protozoa adjacent to the epithelial brush border. Which of the following organisms is the most likely pathogen?

A. Entamoeba histolytica

B. Escherichia coli

C. Giardia lamblia

D. Naegleria fowleri

E. Trichomonas vaginalis

Explanation:

The correct answer is C.

The probable organism is Giardia lamblia, which characteristically infects the small
intestine. When seen in smears from duodenal aspirates, this flagellated organism has a characteristic "face-like" appearance. However, in biopsy specimens, the organism is often caught at an angle, and the characteristic appearance and location is as described in the question stem. Giardia is a common contaminant of water supplies (even in the United States), and patients who ingest the cysts may be asymptomatic or may
occasionally develop prolonged diarrhea or intestinal malabsorption. Some patients with severe disease have low serum IgA or low overall immunoglobulin levels.

Entamoeba histolytica (choice A) usually infects the large intestine and/or liver.

Escherichia coli (choice B) is a bacterial cause of diarrhea.

Naegleria fowleri (choice D) causes meningoencephalitis.

Trichomonas vaginalis (choice E) causes vaginitis.

51.A 60-year-old man presents to the emergency room with severe, tearing pain radiating to the upper back. Over a period of hours the pain moves to the mid back and then involves both flanks. Hematuria develops shortly thereafter. Which of the following renal complications has most likely occurred?

A. Acute glomerulonephritis

B. Bilateral renal infarction

C. Polycystic kidney disease

D. Pyelonephritis

E. Sickle cell crisis

Explanation:

The correct answer is B.

The patient has a dissecting aortic aneurysm that occluded the renal arteries, causing
bilateral renal infarction with flank pain and hematuria. This is a very dangerous complication of dissecting aortic aneurysm, which classically presents as described in the question stem.

Acute glomerulonephritis (choice A) is characterized by hematuria, red cell casts, and often, proteinuria and edema. This syndrome typically develops over days, not hours, and would not be expected to result from dissecting aortic aneurysm.

Polycystic kidney disease (choice C) is a lifelong condition characterized by bilateral polycystic changes in the kidneys. It is associated with hypertension and eventual renal failure.

Pyelonephritis (choice D) is an infection of the kidney that can develop as a complication of bacterial endocarditis, or as a consequence of vesicoureteral reflux, but would not be expected to result from aortic dissection.

Sickle cell crisis (choice E) can cause papillary necrosis of the kidneys with hematuria, but there is no indication that the patient has sickle cell anemia.

52.During a routine preemployment physical examination, an apparently healthy 24-year-old man is found to have increased serum levels of unconjugated bilirubin. Conjugated bilirubin and transaminases are normal. Careful questioning reveals no evidence of recent illnesses. Which of the following is the most likely diagnosis?

A. Crigler-Najjar syndrome

B. Dubin-Johnson syndrome

C. Gilbert's syndrome

D. Rotor syndrome

E. Wilson's disease

Explanation:

The correct answer is C.

Gilbert's syndrome is a relatively common hereditary condition that is so mild, a case
can be made for classifying it as a normal variant rather than a true disease. It produces asymptomatic unconjugated hyperbilirubinemia, and is important to know about to avoid unnecessary diagnostic procedures.

Crigler-Najjar syndrome (choice A) represents several hereditary diseases characterized by severe unconjugated hyperbilirubinemia. The type I form, in particular, can be fatal.

Dubin-Johnson (choice B) and Rotor (choice D) syndromes are relatively mild hereditary conjugated hyperbilirubinemias.

Wilson's disease (choice E) is a serious disorder of copper metabolism associated with hepatic cirrhosis,movement disorder, and pathognomonic gold-colored rings (Kayser-Fleischer rings) in the iris. Diagnosis is based on reduced serum ceruloplasmin (a copper-binding protein), increased hepatic copper content, and increased urinary copper excretion.

53.Which of the following CNS tumors has the worst prognosis and is associated with the shortest survival?

A. Anaplastic astrocytoma (WHO grade III)

B. Glioblastoma multiforme

C. Meningioma

D. Oligodendroglioma

E. Well-differentiated astrocytoma (WHO grade II)

Explanation:

The correct answer is B.

Glioblastoma multiforme (GBM) is the most common and most malignant of the
primary CNS neoplasms. GBM belongs to the category of gliomas called astrocytomas, which originate from neoplastic transformation of astrocytes. There is a continuum in the anaplastic features of astrocytomas, from well-differentiated astrocytoma, characterized by minimal atypia and mitotic activity, to GBM, characterized by a brisk mitotic rate, multifocal necrosis, and extreme nuclear pleomorphism. Since histologic atypia has been
shown to correlate with biologic behavior, several grading systems have been developed to help predict the prognosis of astrocytomas.

The most common of such grading systems (WHO and Saint Anne-Mayo) divide
astrocytomas into four grades, from grade I (benign, well-differentiated) to grade IV (malignant, poorly differentiated).

GBM is grade IV astrocytoma. It is associated with a poor prognosis; most patients die within approximately 12 months of diagnosis.

Well-differentiated astrocytoma (choice E) is basically a synonym for grade II astrocytoma, while anaplastic astrocytoma (choice A) refers to grade III astrocytoma. Survival is better than GBM, although definitive cure is rare, even for well-differentiated astrocytomas.

Meningioma (choice C) is a benign tumor of meningothelial origin that grows from the dura mater, pushing, rather than infiltrating, the underlying brain. Nevertheless, meningiomas may infrequently show histologic features of malignancy and infiltrate the brain, in which case the prognosis is worse (though still better than the dreadful GBM).

Oligodendroglioma (choice D) arises from oligodendroglial cells and usually lacks histologic markers of malignancy (e.g., mitotic activity, nuclear atypia, and necrosis). Owing to its infiltrative pattern of growth,however, complete excision is virtually impossible (remember, there is no basement membrane or other anatomic boundary in the CNS to contain the spread of a glial neoplasm). Thus, oligodendrogliomas recur over
and over following each surgical resection. After each recurrence, these tumors acquire progressively increasing degrees of anaplasia, ultimately becoming similar to GBM. Survival can be very long (7-10 years),but definitive cure is exceptional.

54.Following penicillin treatment for pneumococcal pneumonia, a patient develops a palpable purpuric rash. Biopsy of the rash demonstrates vasculitis with hemorrhage into the skin. The involved arterioles and venules show fibrinoid necrosis and a neutrophilic infiltrate into the wall. Many of the neutrophils are fragmented. Which of the following is
the most likely diagnosis?

A. Allergic granulomatosis and angiitis

B. Giant cell arteritis

C. Leukocytoclastic angiitis

D. Polyarteritis nodosa

E. Wegener's granulomatosus

Explanation:

The correct answer is C.

The correct diagnosis is leukocytoclastic angiitis, which is also known as
hypersensitivity angiitis or “microscopic” polyarteritis nodosa. Affected vessels are usually smaller (arterioles, venules, and capillaries) than those of classic polyarteritis nodosa, and consequently, infarction is less common. The microscopic appearance is that described in the question stem; the fragmentation of the
neutrophils is “leukocytoclasis.” Cases can be either limited to the skin or generalized. Specific antigens (e.g., penicillin) may be suspected as possible triggers, leading to the alternative term “hypersensitivity angiitis.” Removal of the offending antigen in these cases may induce clinical improvement.

Allergic granulomatosis and angiitis (choice A) is an alternative name for Churg- Strauss disease, which affects the respiratory and renal systems.

Giant cell arteritis (choice B) is an alternative name for temporal arteritis, which affects the temporal and ophthalmic arteries.

Polyarteritis nodosa (choice D) primarily affects vessels larger than arterioles.

Wegener's granulomatosus (choice E) affects the respiratory and renal systems.

55.A 60-year-old man suddenly becomes completely blind in one eye, and angiography demonstrates occlusion of the central retinal artery. Which of the following is the most likely cause of the occlusion?

A. Atheroma or embolism

B. Cranial (temporal) arteritis

C. Hypertension

D. Polycythemia vera

E. Tumor

Explanation:

The correct answer is A. The point of this question is that sometimes the obvious explanation is the correct one.
Occlusion of the central retinal artery rapidly causes irreversible blindness with loss of the inner retinal layers.
(The photoreceptor rod and cone cells are maintained by the pigment epithelium.) The site of occlusion is typically just posterior to the cribriform plate. A garden-variety atheroma or embolism is overwhelmingly the most common cause of central retinal artery occlusion.

Despite all of the teaching about the risk of blindness in temporal arteritis (choice B), this disorder causes only 10% of central retinal artery occlusions.

Hypertension (choice C) is more apt to cause bleeding than thrombosis.

Polycythemia vera (choice D) could (rarely) cause occlusion because of increased blood viscosity and a tendency for thrombosis.

Tumor (choice E) might also cause retinal artery thrombosis, but this would be far rarer than atheroma.

56.During a pre-employment physical, a 45-year-old man is noted to have a 3 cm palpable nodule in one lobe of an otherwise normal sized thyroid gland. Needle aspiration of the nodule demonstrates polygonal tumor cells and amyloid, but only very scanty colloid and normal follicular cells. Which of the following is the most likely diagnosis?

A. Follicular thyroid carcinoma

B. Hashimoto's disease

C. Medullary thyroid carcinoma

D. Papillary thyroid carcinoma

E. Thyroid adenoma

Explanation:

The correct answer is C.

The most distinctive clue is the presence of amyloid, which specifically suggests
medullary thyroid carcinoma. This carcinoma is a tumor of the neuroendocrine parafollicular cells of the thyroid,rather than the follicular lining epithelium. These cells produce calcitonin, the precursor protein of which can precipitate, forming bands and nodules of amyloid that appear histologically identical to other forms of amyloid.

The other thing to remember about medullary carcinoma of the thyroid is that it can be a component of multiple endocrine neoplasia (MEN) syndromes type IIa (parathyroid disease, pheochromocytomas, medullary carcinoma) and type IIb (parathyroid disease, pheochromocytomas, medullary carcinoma, mucocutaneous ganglioneuromas, Marfanoid habitus).

Because follicular thyroid carcinoma (choice A) closely resembles normal thyroid tissue, it usually cannot be reliably diagnosed based on fine needle aspiration alone.

Hashimoto's disease (choice B) would show lymphocytes, plasma cells, and macrophages on aspiration.

Papillary thyroid carcinoma (choice D) can be diagnosed by aspiration if papillary clusters are seen.

Thyroid adenoma (choice E) cannot be reliably distinguished from well-differentiated thyroid carcinoma on
aspiration.

57.A 56-year-old alcoholic man is brought in to the emergency room after being found unconscious by his daughter,who called the paramedics. Paramedics report finding the man in a stuporous condition in the bathtub, covered with vomit. On arrival to the emergency room, the man is clammy and his blood pressure is 85/50. Which of the
following conditions is the most likely cause of his hypotension?

A. Acute hemorrhagic pancreatitis

B. Chronic calcifying pancreatitis

C. Chronic obstructive pancreatitis

D. Cystic fibrosis

E. Pancreatic pseudocyst

Explanation:

The correct answer is A.

Acute hemorrhagic pancreatitis is a life-threatening abdominal emergency that is most
often seen in the setting of excessive acute alcohol or food ingestion. In this condition, activated pancreatic enzymes are released into the tissues, where they cause severe local damage to the pancreas, with pain radiating to the back. The enzymes are also released into the blood stream. Shock may result from hemorrhage, activation of bradykinin and related peptides, and/or release of proteolytic and lipolytic enzymes into the circulation. Other systemic manifestations include hypocalcemia, glucose intolerance, and jaundice.

Chronic calcifying pancreatitis (choice B) is seen in chronic alcoholics, but does not cause the dramatic presentation of acute hemorrhagic pancreatitis.

Chronic obstructive pancreatitis (choice C) is seen in gallstone disease.

Cystic fibrosis (choice D) is an inherited disease that usually causes death by age 30.

Pancreatic pseudocyst (choice E) is an acquired loculation of fluid that may be seen after pancreatitis or
trauma.

58.An abdominal x-ray performed on a 54-year-old man demonstrates a large, irregular, calcified mass with multiple broad projections filling one renal pelvis. Which of the following laboratory findings might be expected in this patient?

A. Decreased urine pH

B. Hypercalcemia

C. Hyperuricemia

D. Increased ammonia concentration in the urine

E. Increased cystine concentration in the urine

Explanation:

The correct answer is D.

The patient has a stag-horn calculus. These very large calculi are almost always
composed principally of magnesium ammonium phosphate (often with enough calcium to be radio-opaque) and form in the setting of infection by urea-splitting bacteria such as Proteus.

Increased urine ammonia concentrations are a byproduct of the bacterial metabolism of urea, and tend to increase urine pH (compare with choice A).

Hypercalciuria, with or without hypercalcemia (choice B), is a cause of calcium oxalate stones.

Uric acid stones can be seen in patients with hyperuricemia (choice C) secondary to gout, or in conditions in which a very rapid cell turnover occurs (e.g., leukemias).

Genetically determined defects in the renal transport of amino acids are associated with cystine stones (choice E).

59.Oral examination of a 57-year-old female reveals a 1-cm, flat, white patch on the buccal mucosa. Which of the following diagnoses indicates the greatest likelihood that this lesion will progress to an oral malignancy?

A. Hairy leukoplakia

B. Leukoplakia

C. Lichen planus

D. Oral thrush

E. Squamous papilloma

Explanation:

The correct answer is B.

Leukoplakia is a white plaque on the oral mucosa for which a more specific diagnosis
cannot be rendered. Leukoplakia is often associated with hyperkeratosis and may or may not show dysplastic squamous epithelium. On average, 5% of leukoplakias contain in situ or overt carcinoma.

Hairy leukoplakia (choice A) is an oral infective lesion seen almost exclusively in HIV infection. It is a fluffy, white hyperkeratotic lesion in which a destructive piling up of keratotic squames is seen. Hairy leukoplakia is associated with viral infection, mostly EBV, HPV, and/or HIV. It does not progress to cancer.

Lichen planus (choice C) is a dermatologic condition that may present with white oral plaques. Microscopically,the lesion is characterized by intense lymphocytic infiltration of the dermoepidermal (or mucosal-submucosal) junction with destruction of the basal layer of cells. This inflammatory condition has not been shown to be premalignant.

Oral thrush (choice D) is a superficial candidal infection, typically occurring in the immunosuppressed, or very young. Thrush is an infectious, non-neoplastic disease.

Squamous papilloma (choice E) is a benign human papillomavirus (HPV) infection of the oral mucosa. Typically associated with HPV genotypes 6 and 11, squamous papilloma only rarely progresses to squamous carcinoma.

60.A 71-year-old white male has had polycythemia vera for 12 years. Throughout this period he was treated by phlebotomy alone, and has remained stable. Recently he has noted a dragging sensation in his abdomen, and physical examination reveals massive splenomegaly, palpable to 7 cm below the ribs. This finding may represent
the conversion of polycythemia vera to which of the following hematological disorders?

A. Chronic myeloid leukemia

B. Hairy cell leukemia

C. Myelodysplastic syndrome

D. Myeloid metaplasia with myelofibrosis

E. Waldenström's macroglobulinemia

Explanation:

The correct answer is D.

There are four types of myeloproliferative disorders: chronic myeloid leukemia (CML),
polycythemia vera, myeloid metaplasia with myelofibrosis (MMM), and essential thrombocythemia (which is very rare).

Both CML and polycythemia vera are characterized by hypercellular bone marrow producing all three cell lines. Over a period of years, the bone marrow may burn out, and the marrow space is replaced by fibrosis. In this circumstance, hematopoiesis (including production of neoplastic cells) moves to extramedullary sites,predominantly the spleen and liver. This process is called myeloid metaplasia with myelofibrosis.

Although both polycythemia vera and CML (choice A) can progress to MMM, polycythemia vera does not progress to CML.

Hairy cell leukemia (choice B), a leukemia often associated with massive splenomegaly, is derived from a B lymphocyte, not a myeloid stem cell. It would not progress to a myeloproliferative disorder.

Myelodysplastic syndrome (MDS; choice C) refers to myeloid disorders featuring maturation defects and ineffective hematopoiesis. MDS produces cytopenia and acute leukemias, whereas myeloproliferative disorders produce polycythemia and chronic leukemia.

Waldenström's macroglobulinemia (choice E) is a plasma cell dyscrasia that produces IgM in excess. It derives from plasma cells and, as such, is not a myeloproliferative disease.

61.A 30-year-old male presents to his physician for a pre-athletic physical exam. He has no complaints or significant past medical history. During auscultation of the heart, a faint murmur consistent with aortic regurgitation is heard.Examination of the musculoskeletal system shows decreased range of motion of the hips bilaterally and of the spine in rotation and forward bending. No scoliosis is seen. Examination of the eyes and mouth is unremarkable.His skin is smooth and dry. It is thought that he has a spondyloarthropathy. Which of the following tests would best differentiate this patient's disease from the other spondyloarthropathies?

A. HLA-B27

B. MRI of the femoral head

C. Rheumatoid factor

D. Spinal x-ray

E. Stool culture

Explanation:

The correct answer is D.

This young man has an inflammatory arthropathy involving his hips and spine. The
disease is ankylosing spondylitis, which typically occurs in young men. Ankylosing spondylitis begins at the sacroiliac crests, then moves upward in the spine, causing inflammation and destruction of the posterior elements of the vertebral bodies. The posterior aspects of the vertebrae fuse, reducing the range of motion, and partially taking the weight off the vertebrae, leading to an atrophy osteoporosis. The spinal x-ray will show
fusion of the disks and possible intervertebral disc ossification. Patients may also have associated inflammation and fibrosis of the proximal aorta, leading to aortic regurgitation.

HLA-B27 (choice A) is a class I histocompatibility antigen that has a strong association with ankylosing spondylitis, Reiter's syndrome, psoriatic arthritis, and certain enteropathic arthritides. It is found in all spondyloarthropathies and therefore can not help to distinguish between them.

MRI of the femoral head (choice B) would reveal inflammatory arthritis in that location, but that would not be enough to differentiate ankylosing spondylitis from the other entities. The spinal findings are more characteristic of this disease.

Rheumatoid factor (choice C) is typically negative for all of the spondyloarthropathies, as are other serologic markers of rheumatoid arthritis.

Stool culture (choice E) is not of diagnostic importance in distinguishing these entities. Ankylosing spondylitis is not associated with diarrhea or stool pathogens.

62.A patient consults a physician because of a small lesion on the lips that, on biopsy, proves to be a mucosal neuroma. The patient's mother had medullary carcinoma of the thyroid. In addition to medullary carcinoma of the thyroid, to which of the following conditions would this patient be particularly vulnerable?

A. Gastrinoma

B. Insulinoma

C. Parathyroid adenoma

D. Pheochromocytoma

E. Pituitary adenoma

Explanation:

The correct answer is D.

You should recognize this as a probable case of multiple endocrine neoplasia,
specifically, MEN II b. Features of this autosomal dominant condition include medullary carcinoma of the thyroid, pheochromocytoma, and oral and intestinal ganglioneuromatosis (including mucosal neuromas).

MEN 1 = Parathyroid tumors, pancreatic tumors, and pituitary tumors.
MEN 2a = Medullary thyroid cancers (MTC), pheochromocytoma, and parathyroid tumors.
MEN 2b = Medullary thyroid cancers, pheochromocytoma and neuromas.

Gastrinomas (choice A) and insulinomas (choice B) are found in MEN I.

Parathyroid adenomas (choice C) are found in MEN I and II.

Pituitary adenomas (choice E) are found in MEN I.

63.A 35-year-old woman complains of severe lower abdominal pain, which is worst during menstruation.Laparoscopic examination of the pelvis demonstrates multiple small brown spots on the surface of pelvic structures. Most of these lesions are cauterized, but biopsy of one of the remaining lesions reveals glandular tissue resembling normal endometrium. No cytologic atypia or abnormally shaped glands are seen. Which of the following is the most likely diagnosis?

A. Acute endometritis

B. Adenomyosis

C. Chronic endometritis

D. Endometriosis

E. Metastatic endometrial cancer

Explanation:

The correct answer is D. This benign condition is endometriosis, rather than metastatic endometrial cancer.Endometriosis is defined as a benign growth of endometrium at sites at which it does not normally occur (excluding the myometrium, at which site it is called adenomyosis). Endometriosis is common and is a significant cause of both pain and pelvic scarring. Most of the problems occur because the abnormally located
endometrium responds to hormonal control and may menstruate, producing a very irritating fluid. Endometriosis may apparently be started either by seeding of menstrual cells in the pelvis (entering through the open end of the fallopian tubes) or by metaplasia of mesothelial or other cells (accounting for rare, well-documented cases of endometriosis of bizarre sites such as nasal mucosa or lungs).

Acute endometritis (choice A) is acute inflammation of the endometrium.

Adenomyosis (choice B) is characterized by foci of endometrium deep in the myometrium.

Chronic endometritis (choice C) is chronic inflammation of the endometrium.

Metastatic endometrial cancer (choice E) would be characterized by abnormal glands and cytologic atypia.

64.A 32-year-old man presents to his physician for a routine physical examination. The man admits to recent loss of 10 pounds and occasional fatigue, but ascribes these to increases in his workload. On physical examination there is a 2-3 cm firm, freely moveable, nontender mass in his neck on the right side. Biopsy of the neck mass reveals Reed-Sternberg cells in a mixed inflammatory infiltrate. CT scan reveals marked enlargement of
mediastinal nodes and the presence of a single nodule in his liver. How should his disease be staged?

A. Stage IA

B. Stage IB

C. Stage IIA

D. Stage IIB

E. Stage IIIA

F. Stage IIIB

G. Stage IVA

H. Stage IVB

Explanation:

The correct answer is F.

This is a classic presentation for Hodgkin's disease, a form of lymphoma characterized
by neoplastic proliferation of Reed-Sternberg cells admixed with variable numbers of reactive lymphocytes,neutrophils, and eosinophils. About half the patients have usually painless adenopathy in the neck or supraclavicular area at the time of diagnosis. A minority have constitutional symptoms such as fatigue, weight loss, and night sweats, but these can be important clues. Staging of Hodgkin's disease is based on the extent
of spread and the presence or absence of constitutional symptoms.

The principal stage is determined by location of the tumor:
Stage I indicates that the cancer is located in a single region, usually one lymph node and the surrounding area. Stage I often will not have outward symptoms.
Stage II indicates that the cancer is located in two separate regions, an affected lymph node or organ within the lymphatic system and a second affected area, and that both affected areas are confined to one side of the diaphragm - that is, both are above the diaphragm, or both are below the diaphragm.
Stage III indicates that the cancer has spread to both sides of the diaphragm, including one organ or area near the lymph nodes or the spleen.
Stage IV indicates diffuse or disseminated involvement of one or more extralymphatic organs, including any involvement of the liver, bone marrow, or nodular involvement of the lungs.

A or B: the absence of constitutional (B-type) symptoms is denoted by adding an "A" to the stage; the presence is denoted by adding a "B" to the stage.

65.In which of the following sites is embryonal rhabdomyosarcoma most likely to occur?

A. Gastrointestinal tract

B. Head and neck

C. Kidneys

D. Liver

E. Lungs

Explanation:

The correct answer is B.

Embryonal rhabdomyosarcoma is the most common form of rhabdomyosarcoma and is composed of cells resembling those found in developing muscle, predominately small, round cells. In embryonal rhabdomyosarcomas that protrude into an open space, the malignant cells immediately below the surface may be more densely packed, forming a "cambium" layer.

The head and neck (particularly the nose, nasopharynx,and orbit) is the most frequent site for embryonal rhabdomyosarcoma; other favored sites are the genitourinary tract and the extremities.

66.In which of the following neurodegenerative conditions would you expect to observe the phenomenon known as anticipation?

A. Familial Alzheimer disease (FAD)

B. Familial amyotrophic lateral sclerosis (ALS)

C. Huntington disease

D. Pick disease

E. Progressive supranuclear palsy

Explanation:

The correct answer is C.

Anticipation is a phenomenon in which the phenotype of a disease worsens over
successive generations. This has been observed in families affected by a hereditary disorder because of an expansion of unstable sequences of nucleotide repeats (triplet repeat expansion). Clinical features worsen with each successive generation as the number of triplet repeats increases. Huntington disease is caused by expansion of an unstable CAG repeat in a gene encoding a protein called huntingtin, of unknown function.

All the unstable triplet-repeat disorders identified so far are associated with neurodegenerative conditions. Other examples are fragile X syndrome, myotonic dystrophy, and Friedreich ataxia.

FAD (choice A) comprises 5% to 10% of all cases of Alzheimer disease and is due to autosomal dominant mutations of three different genes: amyloid precursor protein (APP) gene, presenilin-1 gene, and presenilin-2 gene. The e4 allele of the gene encoding apolipoprotein E increases the risk for FAD. Unstable repeat expansion is not a cause of FAD.

The great majority of cases of ALS (choice B)are sporadic. A small subset of familial ALS is caused by mutations in the gene coding for superoxide dismutase on chromosome 21.

Pick disease (choice D) and progressive supranuclear palsy (choice E) are usually sporadic. The genetic alterations of the very few familial cases reported have not been elucidated.

67.A 37-year-old woman is in a serious automobile accident and sustains a closed head injury. She does not immediately seek medical attention, but is brought to the emergency room two hours later by her brother. On physical examination, there is mydriasis and loss of the pupillary light reflex. Several hours later, she is unable to follow a flashlight with her eyes. Which of the following types of herniation is most likely occurring in this patient?

A. Cerebellar tonsils into the foramen magnum

B. Cerebellum upward past the tentorium

C. Cingulate gyrus under the falx

D. Medulla into the foramen magnum

E. Temporal lobe under the tentorium

Explanation:

The correct answer is E.

Head trauma can cause subdural or epidural hematomas that force the medial aspect
of the temporal lobe (uncus) under the free edge of the tentorium cerebelli (an uncal herniation). The resulting compression of the oculomotor (III) nerve characteristically affects the peripheral areas of the nerve, which carry parasympathetic information, before affecting the central areas of the nerve, which carry somatomotor information. Recognizing the development of symptoms related to this damage may permit life-saving medical
or surgical intervention before irreversible damage is done to the brainstem.

Tonsillar herniation into the foramen magnum (choice A) can compress the medulla, causing respiratory failure and death.

Upward herniation of the cerebellum (choice B) is occasionally seen in patients with posterior fossa lesions.

Herniation of the medially located cingulate gyrus under the falx cerebri (choice C), also called subfalcine or cingulate herniation, can be seen with cerebral hemisphere expansion and can compress the anterior cerebral artery.

The medulla (choice D) is normally in the foramen magnum.

68.Following a fall on a basketball court, a 20-year-old man develops a swollen and painful upper arm. Over the next several weeks, the involved area becomes more circumscribed and firm, and then later evolves to a painless,hard, well-demarcated mass. X-ray of the arm at this point would most likely demonstrate which of the following?

A. Benign-appearing bony outgrowths from the humerus

B. Dislocation of the shoulder

C. Flocculent radiodensities surrounding a radiolucent center

D. Healing fracture

E. Malignant-appearing bony outgrowths from the humerus

Explanation:

The correct answer is C.

This is a typical presentation of myositis ossificans, which usually occur in athletic
adolescents and young adults following trauma. Following muscle trauma, an area of damage heals with a fibroblastic proliferation that then ossifies, even though there is no connection to bone.

Flocculent radiodensities surrounding a radiolucent center are seen on x-ray. The lesion can microscopically resemble extraskeletal osteosarcoma, but the clinical history is usually quite different . Simple excision usually cures myositis ossificans.


2 comments:

Unknown said...

Its a pretty impressive about the pathology. Really glad to know. Thanks for the post.
Pancreatitis Treatment Delhi

Florence said...

I started on COPD Herbal treatment from Ultimate Life Clinic, the treatment worked incredibly for my lungs condition. I used the herbal treatment for almost 4 months, it reversed my COPD. My severe shortness of breath, dry cough, chest tightness gradually disappeared. Reach Ultimate Life Clinic via their website at www.ultimatelifeclinic.com I can breath much better and It feels comfortable!

Post a Comment